[obm-l] RE: [obm-l] RE: [obm-l] Dúvida Indução

2012-05-16 Por tôpico Thiago Bersch
Então eu estava tentando fazer mas parava no mesmo ponto, fazia 2+5+8+...+(3n-1)+[(3n-1)+1], chegando aí eu me perco From: joao_maldona...@hotmail.com To: obm-l@mat.puc-rio.br Subject: [obm-l] RE: [obm-l] Dúvida Indução Date: Mon, 14 May 2012 15:24:47 -0300 Vamos dizer que para n respeite

[obm-l] RE: [obm-l] Dúvida Indução

2012-05-14 Por tôpico João Maldonado
Vamos dizer que para n respeite a formula Logo 2+4+6+...+2n=n.(n+1) Somando 2n+2 2+4+6+...+(2n+2=n(n+1)+2n+2=(n+1)(n+2) que respeita a formula Logo se vale para n, vale para n+1 Como vale para 1, vale para 2, e entao para 3, 4, 5... Vale para qualquer natural Tente fazer o segundo agora []s Joao

[obm-l] Re: [obm-l] dúvida em uma afirmação de um vestibular da UEM

2012-05-01 Por tôpico Bernardo Freitas Paulo da Costa
2012/5/1 Vanderlei * vanderma...@gmail.com: Pessoal, a afirmação a seguir é verdadeira ou falsa? Penso que seja verdadeira, porém o gabarito do vestibular diz ser falsa! Preciso de ajuda! Em um plano, existem duas figuras F1  e F2, cujas bases estão sobre uma reta r  do plano, com a seguinte

[obm-l] RES: [obm-l] Re: [obm-l] dúvida em uma afirmação de um vestibular da UEM

2012-05-01 Por tôpico bouskela
...@mat.puc-rio.br] Em nome de Bernardo Freitas Paulo da Costa Enviada em: terça-feira, 1 de maio de 2012 08:23 Para: obm-l@mat.puc-rio.br Assunto: [obm-l] Re: [obm-l] dúvida em uma afirmação de um vestibular da UEM 2012/5/1 Vanderlei * vanderma...@gmail.com: Pessoal, a afirmação a seguir é verdadeira

[obm-l] Re: [obm-l] Re: [obm-l] Dúvida de geometria analítica: É perpendicular, portanto... (?)

2012-03-22 Por tôpico Felipe Blassioli
Muito obrigado. Fiquei perdido porque o livre fez essa passagem do portanto sem fazer referência explícita à operação vetorial e eu não tinha conhecimento algum de álgebra linear, por isso fiquei perdido, mas agora deu pra situar ^^ 2012/3/22 Eduardo Wilner eduardowil...@yahoo.com.br Antes de

[obm-l] Re: [obm-l] Dúvida

2012-03-12 Por tôpico tarsis Esau
a) Fazendo-se a diferença entre (100c+10b+a) - (100a+10b+c) = 396 99c-99a=396 c-a=4 Sendo 2c=a = a=4 e c=8, como a,b,c formando uma PA, temos que b=6 b) A área do triângulo QMN pode ser dada por S1 = [(QP/2)x(QR/2)xsenQ]/2 = S1=(QPxQRxsenQ)/8 A área do triângulo QPR pode ser dada por S2 =

[obm-l] RE: [obm-l] Re: [obm-l] Dúvida

2012-03-12 Por tôpico Vanessa Nunes de Souza
Obrigada!Vanessa Nunes Date: Mon, 12 Mar 2012 17:04:46 -0300 Subject: [obm-l] Re: [obm-l] Dúvida From: tarsise...@gmail.com To: obm-l@mat.puc-rio.br a) Fazendo-se a diferença entre (100c+10b+a) - (100a+10b+c) = 396 99c-99a=396 c-a=4 Sendo 2c=a = a=4 e c=8, como a,b,c formando uma PA, temos

[obm-l] Re: [obm-l] Re: [obm-l] Dúvida de Lógica

2012-02-08 Por tôpico Ralph Teixeira
H nao sei nao. Vou usar C para estah contido e E para pertence a. Concordo que toda inclusao de conjuntos pode ser pensada como uma implicacao (bom, com um quantificador para todo). Afinal: A C B eh o mesmo que dizer para todo x, xEA = xEB Por isso, concordo que a Teoria dos Conjuntos e

[obm-l] Re: [obm-l] Dúvida de Lógica

2012-02-07 Por tôpico Francisco Barreto
Saudações a todos! Seja A o conjunto dos objetos que satisfazem a propriedade r de que 2 3. Seja B o conjunto dos objetos que satisfazem a propriedade s de que Todo brasileiro é desonesto Sabemos que A é o conjunto vazio. O conjunto vazio está contido em qualquer conjunto, incluindo B. Portanto

[obm-l] Re: [obm-l] Dúvida de Lógica

2012-02-07 Por tôpico Ralph Teixeira
Sim, **logicamente**, a frase 23 == Eu sou o papa estah correta. A implicacao logica eh um simbolo DEFINIDO por esta tabela-verdade: p q p = q V V V V F F F V V F F V e, a principio, eh soh isso. Nada em p=q **intrinsicamente** significa causa, efeito, razao ou qualquer coisa

[obm-l] Re: [obm-l] Dúvida de Lógica

2012-02-07 Por tôpico Francisco Barreto
o termo = traduz-se em se-então. Se todos os elementos de A satisfazem r então todos os elementos de A satisfazem s. Que zona! 2012/2/7 Francisco Barreto fcostabarr...@gmail.com Saudações a todos! Seja A o conjunto dos objetos que satisfazem a propriedade r de que 2 3. Seja B o conjunto

[obm-l] Re: [obm-l] Dúvida

2011-11-02 Por tôpico Victor Hugo Rodrigues
Como assim? Acho que falta algo aí. Em 2 de novembro de 2011 17:17, Kleber Bastos klebe...@gmail.com escreveu: Olá grupo, Estou me enrolando nesta prova. Mostre q ∀ nº a/b0, MDC(a,b) = 1, é válido: f(a/b) = f(1)^a/b . -- Kleber.

[obm-l] Re: [obm-l] Re: [obm-l] Dúvida

2011-11-02 Por tôpico Kleber Bastos
É isso mesmo: Mostrar que ∀ nº racional a/b0, M.D.C.(a,b)=1 é válida a sentença: f(a/b)=f(1)^a/b ( f(1) elevado a a/b) Em 2 de novembro de 2011 20:57, Victor Hugo Rodrigues victorhcr.victorh...@gmail.com escreveu: Como assim? Acho que falta algo aí. Em 2 de novembro de 2011 17:17, Kleber

[obm-l] Re: [obm-l] Re: [obm-l] Re: [obm-l] Dúvida

2011-11-02 Por tôpico Joao Maldonado
-feira, 2 de Novembro de 2011 22:21 Assunto: [obm-l] Re: [obm-l] Re: [obm-l] Dúvida É isso mesmo: Mostrar que ∀ nº racional a/b0, M.D.C.(a,b)=1 é válida a sentença: f(a/b)=f(1)^a/b ( f(1) elevado a a/b) Em 2 de novembro de 2011 20:57, Victor Hugo Rodrigues victorhcr.victorh...@gmail.com escreveu

[obm-l] Re: [obm-l] Re: [obm-l] Re: [obm-l] Re: [obm-l] Dúvida

2011-11-02 Por tôpico Kleber Bastos
, MDC(a,b) = 1, é válido: f(a/b) =... -- *De:* Kleber Bastos klebe...@gmail.com *Para:* obm-l@mat.puc-rio.br *Enviadas:* Quarta-feira, 2 de Novembro de 2011 22:21 *Assunto:* [obm-l] Re: [obm-l] Re: [obm-l] Dúvida É isso mesmo: Mostrar que ∀ nº racional a/b0, M.D.C.(a,b

[obm-l] Re: [obm-l] Dúvida

2011-10-24 Por tôpico Hugo Fernando Marques Fernandes
Lucas corre 2/5 da ponte com velocidade de 15 km/h. Sendo p o comprimento da ponte, leva (2/5)p/15 = 2p/75 h para sair da ponte. Pedro corre 3/5 da ponte com velocidade de 15 km/h. Sendo p o comprimento da ponte, leva (3/5)p/15 = 3p/75 h para sair da ponte. A diferença entre o momento em que

[obm-l] Re: [obm-l] Re: [obm-l] Dúvida

2011-10-24 Por tôpico Julio César Saldaña
pedro, portanto su velocidade é 5x15=75. Julio Saldaña -- Mensaje original --- De : obm-l@mat.puc-rio.br Para : obm-l@mat.puc-rio.br Fecha : Mon, 24 Oct 2011 17:08:45 -0200 Asunto : [obm-l] Re: [obm-l] Dúvida Lucas corre 2/5 da ponte com velocidade de 15 km/h. Sendo p o comprimento da ponte

[obm-l] RE: [obm-l] Re: [obm-l] dúvida - teoria dos números

2011-08-05 Por tôpico Rhilbert Rivera
16:00:56 -0700 From: jeffma...@yahoo.com.br Subject: [obm-l] Re: [obm-l] dúvida - teoria dos números To: obm-l@mat.puc-rio.br Tente pensar no pequeno Teorema de Newton, ou se preferir use congruencias.abs De: Marco Antonio Leal marcoantonio_elemen...@hotmail.com Para: obm-l@mat.puc-rio.br Enviadas

[obm-l] Re: [obm-l] dúvida - teoria dos números

2011-07-31 Por tôpico Jefferson Franca
Tente pensar no pequeno Teorema de Newton, ou se preferir use congruencias. abs De: Marco Antonio Leal marcoantonio_elemen...@hotmail.com Para: obm-l@mat.puc-rio.br Enviadas: Quinta-feira, 28 de Julho de 2011 21:39 Assunto: [obm-l] dúvida - teoria dos números

[obm-l] Re: [obm-l] dúvida - teoria dos números

2011-07-31 Por tôpico Jefferson Franca
Desculpe, aliás, devo pedir desculpas a Fermat, troquei o Fermat por Newton, ou seja, use o pequeno teorema de FERMAT. Boa sorte aí De: Marco Antonio Leal marcoantonio_elemen...@hotmail.com Para: obm-l@mat.puc-rio.br Enviadas: Quinta-feira, 28 de Julho de 2011

[obm-l] RE: [obm-l] dúvida - teoria dos números

2011-07-28 Por tôpico João Maldonado
Eu iria por partes, Ex:2^10 = -10 (47) 2^20 = 100 = 6 (47)Logo 2^3.2^20 = 48 = 1 ( 47) Uma outra maneira de fazer é achar todos os resíduos mod 47 de 2^n (muito trabalho) Ainda tem o pequeno teorema de Fermat, que 2^46 = 1 em (mod 47) - (2^23-1)(2^23+1) é divisível por 47, como 47 é

[obm-l] Re: [obm-l] Re: [obm-l] RE: [obm-l] Re: [obm-l] Re: [obm-l] dúvida sobre séries

2011-06-08 Por tôpico Artur Costa Steiner
O critério mais simples para mostrar que a série harmônica diverge talvez seja o baseado no seguinte teorema: Se x_n é uma sequência decrescente de reais tal que Soma x_n converge, então lim n x_ n = 0. (Prove isto) Se x_n = 1/n, x_n decresce para 0 mas lim n x_n = 1, o que mostra que Soma x_n

[obm-l] RE: [obm-l] Re: [obm-l] Re: [obm-l] dúvida sobre séries

2011-06-07 Por tôpico Luís Lopes
Sauda,c~oes, Legal este critério, parece ter sido criado para a série harm. E a esse respeito, o autor da pergunta poderia ler também sobre a constante de Euler. []'s Luís Date: Mon, 6 Jun 2011 23:50:37 -0300 Subject: [obm-l] Re: [obm-l] Re: [obm-l] dúvida sobre séries From

[obm-l] Re: [obm-l] RE: [obm-l] Re: [obm-l] Re: [obm-l] dúvida sobre séries

2011-06-07 Por tôpico Rodrigo Renji
Olá! Então acho bem bacana esse também ( e nem é tão complicado de demonstrar, eu acho ) Esse critério pode ser usado para estudar a convergência de [ SOMA de 1/ k^p ] também pois [ SOMA de 2^k / 2^(kp) ] = [ SOMA de 2^(k (1-p)) ] se 1 - p 0, isto é 1 p a série converge por série

[obm-l] Re: [obm-l] dúvida sobre séries

2011-06-06 Por tôpico Ralph Teixeira
Cuidado: nao confunda o TERMO GERAL de uma serie com a SERIE em si... Na serie SOMATORIO(a_n), o termo geral eh a_n. Mas a serie consiste em SOMAR todos esses a_n. A SEQUENCIA 1/n converge para 0 quando n vai para infinito. 1/n eh o termo geral da serie SOMATORIO(1/n) -- mas nao eh a SERIE. A

[obm-l] Re: [obm-l] dúvida sobre séries

2011-06-06 Por tôpico Bruno França dos Reis
Bom, primeiro vamos deixar claro que é absolutamente impossível que ela convirja para 0. Seja a_n a sequência definida por a_n = 1/n, para todo n = 1. Seja s_n a n-ésima soma parcial da série, isto é, s_n = soma[i = 1 .. n] a_i = soma[i = 1 .. n] 1/i. A soma da sua série é igual a lim[n -- +oo]

[obm-l] Re: [obm-l] dúvida sobre séries

2011-06-06 Por tôpico Victor Seixas Souza
Essa série é a série Harmônica, ela diverge porque a *soma* dos seus termos vai para o infinito. Mais tecnicamente, a soma dos termos pode ficar tão grande quanto se queira aumentando a quantidade de termos. Existe uma prova clássica para iss, feita pelo Nicolau d'Oresme e é a seguinte: S = 1 +

[obm-l] Re: [obm-l] dúvida sobre séries

2011-06-06 Por tôpico Emanuel Valente
Vamo lá... acho q aqui vai ser mais fácil entendre... Desenhe os eixos x e y e vários retangulos juntos com base 1 e de aréa 1, 1/2, 1/4 +... trace a curva 1/x nesse gráfico... vc terá a seguinte relação: Sn = 1 + 1/2 + 1/3 + ... = integral((1/x) dx), de 1 até n+1 ou seja, a soma das áreas dos

[obm-l] Re: [obm-l] Re: [obm-l] dúvida sobre séries

2011-06-06 Por tôpico Rodrigo Renji
Olá! Uma outra maneira ( além da que os colegas enviaram antes), para mostrar que a série não converge, tem um critério de convergência que acho legal, Critério de condensação de Cauchy: Se x_k é uma sequência decrescente de termos positivos ( como é o caso de 1/k ) então a série [ SOMA de

[obm-l] RE: [obm-l] Dúvida em Geometria

2011-06-02 Por tôpico João Maldonado
Bom, sou estudante de ensino medio, logo minha resposta pode estar errada :D Fazendo z1, z2, z3, z4 como os vértices da base e z5 como o vértice da pirâmide/O como o centro da base, O cosseno do ângulo da base é fácil calcular, já que Oz1 = 4sqrt(2) e Oz5 = 2, z1z5 = 6, logo cos(z5z1O) =

[obm-l] Re: [obm-l] Re: [obm-l] Dúvida - OBM Nível Universitário

2011-01-24 Por tôpico Hugo Fernando Marques Fernandes
Oi, Bruna. Pois é, eu já tinha ouvido dizer isso e queria confirmar. É uma pena, mas fazer o que? Regras são regras... Talvez fosse o caso de criar uma categoria nova pra quem já tem diploma, né? Obrigado pela resposta. Hugo. Em 24 de janeiro de 2011 02:20, Bruna Campos

[obm-l] Re: [obm-l] Re: [obm-l] Re: [obm-l] Dúvida - OBM Nível Universitário

2011-01-24 Por tôpico Tiago
Seria uma olimpiada mais dificil ainda de ganhar, haha. 2011/1/24 Hugo Fernando Marques Fernandes hfernande...@gmail.com Oi, Bruna. Pois é, eu já tinha ouvido dizer isso e queria confirmar. É uma pena, mas fazer o que? Regras são regras... Talvez fosse o caso de criar uma categoria nova

[obm-l] Re: [obm-l] Dúvida - OBM Nível Universitário

2011-01-23 Por tôpico Bruna Campos
Hugo, que eu saiba não pode. Só pode participar quem não tem diploma de curso superior :( Em 20/01/11, Hugo Fernando Marques Fernandeshfernande...@gmail.com escreveu: Boa noite. Acabo de ser aprovado para o curso de matemática da UERJ e tenho uma dúvida em relação à OBM de nível

[obm-l] Re: [obm-l] Dúvida - OBM Nível Universitário

2011-01-23 Por tôpico Bruna Campos
PS.: E só até o quarto ano de graduação. Abraços! Em 23/01/11, Bruna Camposbda.cam...@gmail.com escreveu: Hugo, que eu saiba não pode. Só pode participar quem não tem diploma de curso superior :( Em 20/01/11, Hugo Fernando Marques Fernandeshfernande...@gmail.com escreveu: Boa noite.

[obm-l] RE: [obm-l] Re: [obm -l] Dúvida-Geometria ana lítica

2010-10-11 Por tôpico marcone augusto araújo borges
diferente tambem. Obrigado pela sua atenção. Date: Sun, 10 Oct 2010 16:57:11 -0700 From: eduardowil...@yahoo.com.br Subject: [obm-l] Re: [obm-l] Dúvida-Geometria ana lítica To: obm-l@mat.puc-rio.br Pelo menos na geometria euclidiana esse triângulo inexiste: a altura relativa à AB vale 6, logo

[obm-l] Re:[obm-l] Dúvida-Geometria ana lítica

2010-10-11 Por tôpico Eduardo Wilner
Claro! Desculpe a distração anterior. Vc. pode considerar que a mediana só pode medir 3 se for perpendicular à AB. Assim, C será a intersecção do prolongamento de BD com a  paralela à AB (eixo dos x) à uma altura 6, sendo A, do triângulo original (o  que vc. mencionou como reto è o

[obm-l] Re: [obm-l] Dúvida-Geometria ana lítica

2010-10-10 Por tôpico Eduardo Wilner
Pelo menos na geometria euclidiana esse triângulo inexiste: a altura relativa à AB vale 6, logo o pé da referida mediana está à uma altura de 3, logo sua medida tem que ser maior...

[obm-l] Re: [obm-l] dúvida sobre lema de teoria dos número s.

2010-06-05 Por tôpico Bruno França dos Reis
As hipóteses do lema são: (1) m|a (2) n|a (3) (m, n) = 1 -- isto é, m e n são primos entre si A tese é: (m*n) | a Vc aplicou o lema inicialmente para o caso m = 4, n = 5, a = 20. Neste caso as 3 hipóteses estão satisfeitas, então vale a tese: 4*5 | 20. Depois, vc tentou aplicar o lema para o

[obm-l] Re: [obm-l] dúvida sobre lema de teoria dos número s.

2010-06-05 Por tôpico Marcelo Salhab Brogliato
Lucas, veja que 4 e 10 nao sao primos entre si, visto que mdc(4, 10) = 2. Logo, o lema não se aplica. abraços, Salhab 2010/6/5 Lucas Hagemaister lucashagemais...@msn.com Tem-se o lema: *Se [image: m|a] e [image: n|a] entao [image: mn|a] quando [image: m], [image: n] sao primos entre si.*

[obm-l] RE: [obm-l] Re: [obm -l] dúvida sobre lem a de teoria dos núme ros.

2010-06-05 Por tôpico Lucas Hagemaister
10 isso não ocorre. From: bfr...@gmail.com Date: Sat, 5 Jun 2010 13:46:07 -0300 Subject: [obm-l] Re: [obm-l] dúvida sobre lema de teoria dos números. To: obm-l@mat.puc-rio.br As hipóteses do lema são: (1) m|a (2) n|a (3) (m, n) = 1 -- isto é, m e n são primos entre si A tese é: (m*n

[obm-l] Re: [obm-l] RE: [obm-l] Re: [obm-l] dúvida sobre le ma de teoria dos números.

2010-06-05 Por tôpico Bernardo Freitas Paulo da Costa
2010/6/5 Lucas Hagemaister lucashagemais...@msn.com  Hum... Entendi. Obrigado! O que mais ou menos o lema quer dizer é o seguinte: Sempre que termos m|a e n|a, onde mn|a, m e n serão primos entre si. Tivermos, para não assassinar o português. E não, cuidado com a ordem das implicações. A e B =

[obm-l] Re: [obm-l] Dúvida sobre logaritmos

2010-05-19 Por tôpico Adalberto Dornelles
Olá Marcelo, log de 81 na base a = 4. PRIMEIRA DÚVIDA A seguir é escrito a = - 3 (não convém) e é confirmada a resposta a = 3. A pergunta é: como explicar este não convém ? Porque não convém ? Porque, por definição, a base do logarítmo é positiva e diferente de 1. SEGUNDA DÚVIDA

[obm-l] Re: [obm-l] Dúvida inequações

2010-04-19 Por tôpico Adalberto Dornelles
Olá Rodrigo, No livro do guidorizzi é dito que: 3x-1/x+2 = 5 não é equivalente a 3x-1 = 5(x+2) (A) porém, 3x-1/x+2 = 5 = 3x-1 = 5(x+2). (B) Não entendi a razão... Bom, em primeiro lugar, leia com atenção o texto entre as duas equações. Nele diz que (A) é verdadeira SE x -2 == x+2

[obm-l] Re: [obm-l] Dúvida inequações

2010-04-17 Por tôpico Bernardo Freitas Paulo da Costa
Rodrigo, é bem simples, mas acho que precisa de um pouco mais de contexto. Repare que -3 -2, mas 3 2. Portanto, quando você vai simplificar frações em inequações (e não igualdades), você tem que prestar atenção se você está multiplicando por um fator positivo ou negativo, que é o que o

[obm-l] Re: [obm-l] Dúvida

2010-01-15 Por tôpico cgomesmat
Pedro estou com pouco tempo para escrever agora...mas está feita no vol 2 do livro A Matemática do ensino médio do impa..vê lá é fácil de entender e bem legal!   Valew, cgomesEm 13/01/2010 14:29, Pedro Costa npc1...@gmail.com escreveu: Oi pessoal, poderiam me ajudar nesta questão:  

[obm-l] Re: [obm-l] Dúvida

2010-01-13 Por tôpico Bruno França dos Reis
Quais três segmentos? -- Bruno FRANÇA DOS REIS msn: brunoreis...@hotmail.com skype: brunoreis666 tel: +33 (0)6 28 43 42 16 http://brunoreis.com GPG Key: http://brunoreis.com/bruno-public.key e^(pi*i)+1=0 2010/1/13 Pedro Costa npc1...@gmail.com Oi pessoal, poderiam me ajudar nesta

[obm-l] Re: [obm-l] Dúvida

2010-01-13 Por tôpico Bernardo Freitas Paulo da Costa
Pedro, seja mais preciso. Quais três segmentos? Note que pondo dois pontos X e Y num segmento, você cria AX, AY, XY (ou YX), XB, YB, além do AB inicial... Além disso, diga também como a gente vai escolher os dois pontos... uniformemente ? independentemente ? ... O maior problema deste problema é

[obm-l] RES: [obm-l] Re: [obm-l] Dúvida

2010-01-13 Por tôpico Pedro Costa
partes dessa região. O enunciado comprometeu a questão e posso usar tamanho “a”. De: owner-ob...@mat.puc-rio.br [mailto:owner-ob...@mat.puc-rio.br] Em nome de Ralph Teixeira Enviada em: quarta-feira, 13 de janeiro de 2010 15:27 Para: obm-l@mat.puc-rio.br Assunto: [obm-l] Re: [obm-l] Dúvida

[obm-l] Re: [obm-l] RES: [obm-l] Re: [obm-l] Dúvida

2010-01-13 Por tôpico Ralph Teixeira
-rio.br *Assunto:* [obm-l] Re: [obm-l] Dúvida Tecnicamente, o problema não está bem formulado, pois não diz exatamente COMO os pontos X e Y são escolhidos. Isto dito, uma interpretação usual é esta aqui: http://www.mat.puc-rio.br/~obmlistas/obm-l.200706/msg00182.html Abraço, Ralph

[obm-l] Re: [obm-l] Dúvida sobre estimadores tendenciosos e não-tendenciosos

2009-11-16 Por tôpico Ralph Teixeira
Oi, Marcelo. Não sou expert nisso não, mas eu faria do mesmo jeito que você fez... Em primeiro, E(D)=0,65... Achei E(u1)=0,31060,65, então o estimador u1 é tendencioso. Achei E(u2)=0,77500,65, então o estimador u2 é tendencioso. Agora, se D1D2 introduz uma condição que faz com que E(D1-D2 |

[obm-l] Re: [obm-l] Re: [obm-l] Dúvida sobre estimadores te ndenciosos e não-tendenciosos

2009-11-16 Por tôpico Marcelo Salhab Brogliato
Olá, Ralph, chegamos aos mesmos valores e conclusões. Entendi perfeitamente o problema dos 100% de amostras 0, hehehe. Muito obrigado pela resposta, grande abraço, Salhab 2009/11/16 Ralph Teixeira ralp...@gmail.com Oi, Marcelo. Não sou expert nisso não, mas eu faria do mesmo jeito que você

[obm-l] Re: [obm-l] Dúvida sobre limites

2009-10-28 Por tôpico Luiz Paulo
Podemos ver Tn da seguinte forma: T(n)=1+1/[1+1/n]*sen(npi/2). Tomando n=2k (k inteiro) vemos que daí teremos sen(kpi) que fica sendo zero. Tomando n=2k+1(k inteiro) teremos sen[(2k+1)pi/2] que oscila entre -1 ou 1 dependendo do k. Daí tomando k tendendo ao infinito vemos que o termo em sen

[obm-l] Re: [obm-l] Re: [obm-l] Dúvida sobre limites

2009-10-28 Por tôpico Paulo Barclay Ribeiro
Paulo paulolui...@yahoo.com.br escreveu: De: Luiz Paulo paulolui...@yahoo.com.br Assunto: [obm-l] Re: [obm-l] Dúvida sobre limites Para: obm-l@mat.puc-rio.br Data: Quarta-feira, 28 de Outubro de 2009, 14:14 Podemos ver Tn da seguinte forma: T(n)=1+1/[1+1/n]*sen(npi/2). Tomando n=2k (k inteiro

[obm-l] Re: [obm-l] Dúvida

2009-08-05 Por tôpico Bruno França dos Reis
Vc pode dizer isso isso. Só reescrevendo sua afirmação: x^2 = sqrt(2^2) E isso é exatamente o mesmo que dizer: x^2 = sqrt(4) Que por sua vez é o mesmo que: x^2 = 2 Finalmente, vc pode dizer, ainda mantendo relação de equivalência com a primeira expressão, x = -sqrt(2) ou x = +sqrt(2) Apenas

[obm-l] Re: [obm-l] RE: [obm-l] Re: [obm-l] dúvida de inter pretação

2009-05-14 Por tôpico Marcelo Costa
-l@mat.puc-rio.br *Subject:* [obm-l] Re: [obm-l] dúvida de interpretação Olá Marcelo Numa fração os termos são necessariamente números inteiros. Mas uma fração pode representar inúmeras coisas: um número, uma divisão, uma RAZÃO etc. A Razão na verdade é uma comparação entre duas

[obm-l] Re: [obm-l] dúvida de interpretação

2009-05-12 Por tôpico Palmerim Soares
Olá Marcelo Numa fração os termos são necessariamente números inteiros. Mas uma fração pode representar inúmeras coisas: um número, uma divisão, uma RAZÃO etc. A Razão na verdade é uma comparação entre duas quantidades, feita por meio da divisão entre essas quantidades, as quais podem ser ou não

[obm-l] RE: [obm-l] Re: [obm-l] dúvida d e interpretação

2009-05-12 Por tôpico Albert Bouskela
To: obm-l@mat.puc-rio.br Subject: [obm-l] Re: [obm-l] dúvida de interpretação Olá Marcelo Numa fração os termos são necessariamente números inteiros. Mas uma fração pode representar inúmeras coisas: um número, uma divisão, uma RAZÃO etc. A Razão na verdade é uma comparação entre duas

[obm-l] Re: [obm-l] Dúvida em questão de Raciocínio

2009-02-03 Por tôpico João Luís
Não fiz ainda, mas creio que é uma simples questão de considerar cada fazenda como sendo um conjunto cujos elementos são o tipo de animal que é criado lá. Então afirmativas como os animais comuns às fazendas Alfa e Beta são somente cães e bois , quer dizer que a intersecção dos conjuntos Alfa

Re: [obm-l] Re: [obm-l] Dúvida em questão de Raciocínio

2009-02-03 Por tôpico Carlos Nehab
Oi, Joo, Foi numa prova da ANPAD. Em geral boas questes... http://www.anpadcurso.com/provas_anteriores/rl/PROVA_TESTE_ANPAD_RL-fev_08.pdf Abraos, Nehab Joo Lus escreveu: No fiz ainda, mas creio que uma simples questo de considerar cada fazenda como sendo um conjunto cujos

[obm-l] Re: [obm-l] Re: [obm-l] Dúvida em questão de Racio cínio

2009-02-03 Por tôpico Ralph Teixeira
Fiz como o João Luís falou, deu certo: faça um diagrama de Venn, preencha de dentro para fora. Chamando os conjuntos de A, B e C, (chamo e de interseção, ou de união): A e B e C = (A e B) e (A e C) = {Cão} (usando II e IV) A e B e (não C) = (A e B) - (A e B e C) = {Boi} (II) A e (não B) e C = (A

[obm-l] Re: [obm-l] Re: [obm-l] Dúvida em questão de R aciocínio

2009-02-03 Por tôpico João Luís
Obrigado Nehab - Original Message - From: Carlos Nehab To: obm-l@mat.puc-rio.br Sent: Tuesday, February 03, 2009 11:27 AM Subject: Re: [obm-l] Re: [obm-l] Dúvida em questão de Raciocínio Oi, João, Foi numa prova da ANPAD. Em geral boas questões... http

[obm-l] Re: [obm-l] Re: [obm-l] Dúvida em questão de R aciocínio CORRIGINDO

2009-02-03 Por tôpico João Luís
: Tuesday, February 03, 2009 11:39 AM Subject: Re: [obm-l] Re: [obm-l] Dúvida em questão de Raciocínio CORRIGINDO Corrigindo: Foi num teste de um curso preparatório para a Anpad. João, deixa de ser preguiçoso e acredite no Google... :-) Foi o que fiz... Nehab Carlos Nehab escreveu

Re: [obm-l] Re: [obm-l] Dúvida em questão de Raciocínio CORRIGINDO

2009-02-03 Por tôpico Carlos Nehab
Corrigindo: Foi num teste de um curso preparatrio para a Anpad. Joo, deixa de ser preguioso e acredite no Google... :-) Foi o que fiz... Nehab Carlos Nehab escreveu: Oi, Joo, Foi numa prova da ANPAD. Em geral boas questes...

[obm-l] Re: [obm-l] Dúvida de lógica

2009-01-23 Por tôpico Denisson
Fábio a linguagem natural gera essas ambiguidades. Portanto em escritos formais é conveniente definir o que você quer dizer (isso mesmo, qualquer definição vale desde que seja consistente e não seja contraditória) e não deixar a cargo de interpretações. No contexto computacional eu usaria

Re: [obm-l] Re: [obm-l] dúvida simples - valor de aderência

2009-01-16 Por tôpico Carlos Silva da Costa
obrigado a todos que me responderam. agora ficou bem claro, 2009/1/16 Artur Costa Steiner artur_stei...@yahoo.com Veja, explicitamente, os termos desta sequencia sao 1,1,2,1/2... A sequencia diverge. Tem uma subseq. que vai para oo e outra que converge para 0, de fato unico ponto de

[obm-l] Re: [obm-l] dúvida simples - valor de aderência

2009-01-15 Por tôpico Artur Costa Steiner
Veja, explicitamente, os termos desta sequencia sao 1,1,2,1/2... A sequencia diverge. Tem uma subseq. que vai para oo e outra que converge para 0, de fato unico ponto de aderencia. Voce esta confundindo, x_2n nao eh a a serie harmonica, nao hah somas. Eh apenas a seq. dos inversos dod naturais,

[obm-l] Re: [obm-l] Dúvida

2008-10-27 Por tôpico João Luís
O problema pede que o número contenha o algarismo 1 E o algarismo 2. Então não tem essa história de só com o 1, nem só com o 2. Sendo assim, temos: Os números formados deverão ter os algarismos 1 e 2, e mais dois algarismos que pertençam ao conjunto {3,4,5,6}. O número de maneiras de escolher

[obm-l] Re: [obm-l] Dúvida persistente

2008-09-17 Por tôpico João Luís
Agora, meu caro Robÿfff, você já tem subsídio pra resolver essa questão. Basta ler os comentários já postados sobre ela, quando ela estava incompleta Um abraço, João Luís - Original Message - From: Robÿe9rio Alves To: obm-l@mat.puc-rio.br Sent: Wednesday, September

[obm-l] Re: [obm-l] Dúvida com questão

2008-09-16 Por tôpico João Luís
Bom, faltou um símbolo de desigualdade no primeiro membro (antecedente) dessa bicondicional: [(x^2+x+1)/(x-2)] 3. Mas de qualquer forma, a afirmativa é falsa, já que o sinal da desigualdade vai mudar de sentido conforme o sinal do termo de primeiro grau (x-2): se (x-2) 0, a segunda

Re: [obm-l] Re: [obm-l] Dúvida com questão

2008-09-16 Por tôpico Bouskela
João Luís: Sua solução está correta! Entretanto repare que ela (sua solução) está correta apenas porque x^2+x+1 é positivo (maior do que 0) para qualquer que seja x real. É, então, necessário, no âmbito da sua solução, demonstrar isto: x^2+x+1 0 para qualquer que seja x real. Sds., AB

[obm-l] Re: [obm-l] Re: [obm-l] Dúvida com questão

2008-09-16 Por tôpico João Luís
- From: Bouskela To: obm-l@mat.puc-rio.br Sent: Tuesday, September 16, 2008 11:46 AM Subject: Re: [obm-l] Re: [obm-l] Dúvida com questão João Luís: Sua solução está correta! Entretanto repare que ela (sua solução) está correta apenas porque x^2+x+1 é positivo (maior do que 0) para

[obm-l] RES: [obm-l] Re: [obm-l] Re: [obm-l] Dúvida com questão

2008-09-16 Por tôpico Bouskela
] _ De: [EMAIL PROTECTED] [mailto:[EMAIL PROTECTED] Em nome de João Luís Enviada em: terça-feira, 16 de setembro de 2008 13:01 Para: obm-l@mat.puc-rio.br Assunto: [obm-l] Re: [obm-l] Re: [obm-l] Dúvida com questão Sim, é verdade. Ficou incompleto mesmo. O que acontece é

[obm-l] RE: [obm-l] Dúvida em trigonometria e nos Complexos

2008-07-22 Por tôpico victorcarlos
Olá Paulo, 1) Para o primeiro , você pode usar a relação para o sen2x e desenvolver ; no entanto acredito ficar mais simples se utilizar a relaçãosen2x = 2t/(1+t^2) onde t =tanx e estudar a desigualdade , ok ? 2) para o segundo , pense assim :no plano de Argand-Gauss , o

[obm-l] Re: [obm-l] RE: [obm-l] Dúvida em trigonomet ria e nos Complexos

2008-07-22 Por tôpico Paulo Mello
PROTECTED] Assunto: [obm-l] RE: [obm-l] Dúvida em trigonometria e nos Complexos Para: obm-l@mat.puc-rio.br, obm-l@mat.puc-rio.br Data: Terça-feira, 22 de Julho de 2008, 15:59 Olá Paulo, 1) Para o primeiro , você pode usar a relação para o sen2x e desenvolver ; no entanto acredito ficar mais

[obm-l] Re: [obm-l] Dúvida em trigonometria e nos Complexos

2008-07-22 Por tôpico Bernardo
Caro colega. Essas questões são de uma avaliação de um curso de atualização de professores do CEDERJ, feito a distância. Acho que se alguém colocar as soluções pra vc aqui, perderá todo o sentido do curso. - Original Message - From: Paulo Mello To: obm-l@mat.puc-rio.br Sent:

[obm-l] Re: [obm-l] DÚVIDA CRUEL

2008-05-01 Por tôpico João Luís
Acabou de ser lançado o volume 4 da coleção, com todos os enunciados e todas as soluções dos exercícios propostos nos primeiros três volumes! Abraços, João Luís - Original Message - From: Pedro Júnior To: obm-l@mat.puc-rio.br Sent: Wednesday, April 23, 2008 11:13 PM Subject:

Re: [obm-l] Re: [obm-l] Dúvida - Teoria dos Anéis

2007-10-01 Por tôpico jones colombo
Realmente (Z,+,.) é um domínio de integridade, ou seja, é um anel com unidade tal que ab=0 implica que a=0 ou b=0. Por outro lado não é um corpo pois nem todo elemento possui inverso multiplicativo por exemplo o número inteiro 3. Não existe nenhum número inteiro b tal que 3b=1. t+ Jones On

[obm-l] Re: [obm-l] Dúvida - Teoria dos Anéis

2007-09-30 Por tôpico Benedito
(Z,+, .) é um anel de integridade? É um corpo? Benedito - Original Message - From: Claudinei - Trix To: obm-l@mat.puc-rio.br Sent: Saturday, September 29, 2007 1:07 PM Subject: [obm-l] Dúvida - Teoria dos Anéis Há um lema que diz o seguinte: Um anel de integridade

Re: [obm-l] Re: [obm-l] Dúvida ( área m ínima )

2007-05-17 Por tôpico cleber vieira
Valeu Rafael Muito obrigado. __ Fale com seus amigos de graça com o novo Yahoo! Messenger http://br.messenger.yahoo.com/

[obm-l] Re: [obm-l] Dúvida ( área mínima )

2007-05-16 Por tôpico rgc
Oi Se o triangulo é retangulo entao a area S = AB*AC/2. Seja D o ponto que a reta t corta a reta r e E o ponto que t corta s. Faça tambem F o ponto que AC intercepta r. Vou chamar teta de x pra facilitar. Se x é o angulo que AC forma com a reta s, (AFD) = x (porque r e s são paralelas). Veja

[obm-l] Re: [obm-l] Dúvida Cálculo - Reta Tangente

2007-05-04 Por tôpico rgc
Oi Derivando a equação voce tem y' = e^x. Se a reta passa pela origem o ponto (0,0) está na reta. Então precisamos de um ponto P pertencente à curva e que a reta que passa por P e por (0,0) tangencie e^x em P. Como P está na curva fazemos P = (a, e^a). Assim a inclinação da reta será (e^a -

[obm-l] RE: [obm-l] Dúvida Cruel!

2006-11-16 Por tôpico Ralph Teixeira
Infelizmente, nao estah correto -- voce nao pode subtrair desigualdades Note: 20 21 e 1 3, entao 19 18 ?? ---///--- Sem calculo, acho que sei achar as raizes INTEIRAS, nao sei... Para as reais, tenho uma solucao *com* calculo: Seja f(y)=y^x (onde x eh constante!) onde y0. A equacao eh

[obm-l] Re:[obm-l] Dúvida Cruel!

2006-11-15 Por tôpico claudio\.buffara
-- Cabeçalho original --- De: [EMAIL PROTECTED] Para: obm-l@mat.puc-rio.br Cópia: Data: Wed, 15 Nov 2006 08:37:16 -0300 (ART) Assunto: [obm-l] Dúvida Cruel! Pessoal como faço pra resolver essa equação? Encontre todas as soluções reais da equação: 2x + 5x = 3x + 4x

Re: [obm-l] Re:[obm-l] Dúvida Cruel!

2006-11-15 Por tôpico Ronaldo Alonso
A solução do Cláudio baseia-se em propriedades das exponenciais que podem ser verificadas por inspeção. Basicamente para resolver ele inicialmente checou que x=0 e x=1 eram soluções (alguém pode rapidamente fazer isso em uma olimpíada, embora é preciso ter alguma intuição anterior, ou

[obm-l] RE: [obm-l] dúvida reincidente

2006-10-23 Por tôpico Luís Lopes
Sauda,c~oes, Oi Orlando, Este problema já foi resolvido. Mostre que n/2 = 16,1 = (2+n)/2 e conclua que n=31 e x (elemento suprimido) = 13. []'s Luís From: Orlando Onofre Filho [EMAIL PROTECTED] Reply-To: obm-l@mat.puc-rio.br To: obm-l@mat.puc-rio.br Subject: [obm-l] dúvida reincidente

[obm-l] Re: [obm-l] Dúvida conceitual ( ou de inglês )

2006-10-17 Por tôpico Gabriel Rovina
O significado é "Sendo "a" e 6 primos entre si..." Acho que está faltando alguma parte não? O * (divisível) não aparece no enunciado... [ ]'s Gabriel - Original Message - From: Ramon Gondim To: obm-l@mat.puc-rio.br Sent: Tuesday, October 17, 2006 1:48 PM

Re: [obm-l] Re: [obm-l] Dúvida conceitual ( ou de inglês )

2006-10-17 Por tôpico Ramon Gondim
Então O enunciado é provar que se a for relaticamente primo em relação a 6, (a^2-1) é divisivel por 24 ( análogo a ( a^2-1 ) : 24 ) A minha dúvida era conceitual mesmo e já foi explicado, Vlw ricardo Em 17/10/06, Gabriel Rovina [EMAIL PROTECTED] escreveu: O significado é Sendo a e 6 primos

[obm-l] Re: [obm-l] Dúvida logaritmo

2006-09-23 Por tôpico Marcelo Salhab Brogliato
Olá.. ln (1+h/x)^1/h = [ ln(1+h/x) ] / h = [ ln(x+h) - ln(x) ] / h abracos Salhab - Original Message - From: Douglas Alexandre To: obm-l@mat.puc-rio.br Sent: Saturday, September 23, 2006 4:31 PM Subject: [obm-l] Dúvida logaritmo Caros colegas como mostro que

[obm-l] Re: [obm-l] dúvida

2006-08-20 Por tôpico Ojesed Mirror
Tem um erro na sua substituição da raiz, coloque 81 no lugar do segundo 27. - Original Message - From: cleber vieira To: obm-l@mat.puc-rio.br Sent: Sunday, August 20, 2006 7:49 PM Subject: [obm-l] dúvida Olá amigos gostaria de saber da opinião de vocês sobre

Re: [obm-l] Re: [obm-l] Re:[obm-l] dúvida (Q uadriláteros)

2006-05-05 Por tôpico cleber vieira
Valeu claudio, a idéia de fazerDO =OX e daí provar que X coincide com H foi um xeque-mate no problema, parabéns e muito obrigado pela sua resolução. Abraços Cleber Abra sua conta no Yahoo! Mail - 1GB de espaço, alertas de e-mail no celular e anti-spam realmente eficaz.

[obm-l] RE: [obm-l] dúvida fatorial

2006-04-11 Por tôpico Paulo Santa Rita
viva e real da Matematica. E tambem, parece-me, o que nos da maior motivacao ... Um Abraco a Todos ! Paulo Santa Rita 3,2010,110406 From: Ronaldo Luiz Alonso [EMAIL PROTECTED] Reply-To: obm-l@mat.puc-rio.br To: obm-l@mat.puc-rio.br Subject: [obm-l] Re: [obm-l] RE: [obm-l] dúvida fatorial Date

[obm-l] Re: [obm-l] Re: [obm-l] dúvida fatorial

2006-04-04 Por tôpico Ronaldo Luiz Alonso
Qualquer valor diferente de um atribuído por convenção estaria negando a definição de fatorial. SE considerarmos a interpretação de fatorial como número de bijeções de um conjunto com n elementos em um conjunto com n elementos e SE considerarmos a definição de números binomiais em termos de

Re: [obm-l] Re: [obm-l] dúvida fatorial

2006-04-04 Por tôpico Rogerio Ponce
Olá Mirror, se sua definicao estiver correta, entao suas 2 deducoes, assim como sua conclusao final, estao erradas. Erro da 1a "deducao" : voce esta' querendo concluir qual o valor de um termo (1!) a partir de um termo de ordem mais alta (2!). Mas de onde voce tirou o valor do fatorial de 2 ?!

[obm-l] Re: [obm-l] dúvida fatorial

2006-04-03 Por tôpico Ronaldo Luiz Alonso
Dá uma olhada nisso: http://mathworld.wolfram.com/GammaFunction.html http://en.wikipedia.org/wiki/Gamma_function Talvez sane sua dúvida :) - Original Message - From: reginaldo.monteiro To: obm-l Sent: Monday, April 03, 2006 9:49 AM Subject: [obm-l] dúvida fatorial

[obm-l] RE: [obm-l] dúvida fatorial

2006-04-03 Por tôpico Paulo Santa Rita
Ola Reginaldo e demais colegas desta lista ... OBM-L, ( estou escrevendo sem acentos ) Porque e conveniente ... Este postulado ( 0!=1 ) e consistente com outras crencas e as implicacoes dele sao uteis na pratica. De fato : 1) ACREDITANDO que a FUNCAO GAMA e a generalizacao do conceito de

[obm-l] Re: [obm-l] RE: [obm-l] dúvida fatorial

2006-04-03 Por tôpico Ronaldo Luiz Alonso
euclidianas são um exemplo prático deste caso. WHAT I CAN'T CREATE I CAN'T UNDERSTAND -- RICHARD FEYNMAN. - Original Message - From: Paulo Santa Rita [EMAIL PROTECTED] To: obm-l@mat.puc-rio.br Sent: Monday, April 03, 2006 12:29 PM Subject: [obm-l] RE: [obm-l] dúvida fatorial Ola

[obm-l] Re: [obm-l] dúvida fatorial

2006-04-03 Por tôpico Ojesed Mirror
Por definição n! = n*(n-1)! para n natural maior que 1. Se fizermos n=2 deduzimos que 1!=1 Se fizermos n=1 deduzimos que 0!=1 Então, 0! e 1! são iguais a um por extensão/conseqüência da definição de fatorial e não por convenção. Qualquer valor diferente de um atribuído por convenção estaria

[obm-l] Re: [obm-l] dúvida sobre notação

2006-03-21 Por tôpico João Luís Gomes Guimarães
Minha opinião é que, matematicamente não faz diferença mesmo não, já que o triângulo é o mesmo não importando a ordem que você escreve os vértices. Agora, eu acho altamente recomendável que se respeite a correspondência de vértices na hora de escrever a semelhança. Assim, quando se escreve

[obm-l] Re: [obm-l] Dúvida em Lógica

2006-02-25 Por tôpico Rhilbert Rivera
Antes de mais nada, obrigado ao professor Nicolau pela resposta. Fiquei mais tranquilo. Realmente pela tabela verdade, temos dois casos em que a falta de parênteses causa dúvidas. Minha dúvida veio do fato de que alguns os livros colocam uma ordem deprecedência para os conectivos, como por

[obm-l] Re: [obm-l] Dúvida em Lógica

2006-02-25 Por tôpico Rhilbert Rivera
Prof. Nicolau, tinha esquecido de comentar, mas pesquisando na internet sobre essa questão de conectivos lógicos encontrei uma apostila da PUC-MGno endereço http://www.inf.pucminas.br/professores/cruz/apostila/capitulo03.pdf ou então em

<    1   2   3   4   >